Revisão de Limites

Daniel Miranda
UFABC

Sumário

Introdução

  • Pressione ESC para ver um panorama dos slides.

  • As resoluções dos exemplos e as demonstrações dos teoremas podem ser vistas clicando na seta para baixo, quando esta estiver disponível.

  • Use as setas do teclado para navegar mais facilmente.

  • Em telas touch, deslize os dedos para trocar de slides.

Limites e Continuidade de Funções

‘'It has long been an axiom of mine that the little things are infinitely more important.’' - Sherlock Holmes, in A Case of Identity, Arthur Conan Doyle

Intuições sobre Limite

O conceito de limite de uma função num ponto $a$ descreve o comportamento dessa função em valores próximos de $a$, mas diferentes de $a$.

Descrição Informal de Limite:
Dizemos que o limite da função $f(x)$ é $L$ quando $x$ tende a $a$ se a função $f(x)$ torna-se arbitrariamente próxima de $L$ quando $x$ está suficientemente próximo de $a$, mas diferente de $a$. Denotaremos tal fato por:
(1)
\[\limitex{a} f(x)=L \]

Como o limite com $x$ tendendo a $a$ de $f(x)$ descreve o comportamento da função $f$ para valores próximo a $a$, mas diferentes de $a$, assim uma exigência natural a ser imposta sobre a função $f$ é que esta esteja definida ao menos num intervalo contendo $a$, exceto possivelmente no próprio ponto $a$.

Os gráficos da Figura 1 mostram três exemplos de funções para os quais os limites existem e são $L$. No primeiro caso a função $f$ está definida em $a$, e $f(a)=L$, na segunda a função $g$ não está definida em $a$ e na terceira apesar da função estar definida em $a$ temos que $h(a)\neq L$. Já os gráficos da Figura 2 ilustram duas situações nas quais o limite em $a$ não existe.

limite1


Figura 1. 

limite2


Figura 2. 

Definição de Limite

Para formalizar a descrição informal de limite que apresentamos na seção anterior, um passo importante é formalizar o conceito de próximo.

Dizemos que um ponto $y$ é uma aproximação de $a$ com erro menor que $\delta$ se $y$ satisfaz $\left| y-a\right|<\delta$, ou seja se $y\in \left(a-\delta,a+\delta\right)$. De modo análogo, dizemos que a função $f(x)$ é uma aproximação de $L$ com erro menor que $\eps$ para $L$ para valores de $x$ suficientemente próximos de $a$, se para $y: \abs{y-a}<\delta$ então $\left|f(x)-L\right|<\eps$.

De posse desses conceitos, podemos reescrever a definição de limite como:

Definição [Limite ].
Seja $f$ uma função definida num intervalo aberto contendo $a$, exceto possivelmente no próprio ponto $a$ e seja $L$ um número real. Dizemos que o limite de $f(x)$ é $L$ quando $x$ tende a, denotado por:

(2)
\[\limitex{a}f(x)=L, \]

se para todo $\eps>0$ existe um $\delta>0$ tal que

(3)
\[\text{se } 0<\abs{x-a}<\delta \text{ então } \abs{f(x)-L}<\eps. \]

limite3


Figura 3. 

A notação $\limitex{a} f(x)=L$ significa que o limite existe e é igual a $L.$

Exercício. Mostre a partir da definição de limite que $\limitex{a}c=c$

Resolução. Como dito anteriormente para demostrar um limite temos que estimar $\abs{f(x)-L}$ numa vizinhança de $a$.

Nesse caso temos que $\abs{f(x)-L}=\abs{c-c}=0$, independente dos valores de $x$. Ou seja, para qualquer $\delta$ se $0<\abs{x-a}<\delta$ então $\abs{f(x)-L}=\abs{c-c}=0<\eps$

Exercício. Mostre a partir da definição de limite que $\limitex{a}x=a$

Resolução. Dado $\eps>0$, como:

(4)
\[\abs{f(x)-L}=\abs{x-a} \]

Podemos escolher o valor de $\delta$, fazendo $\delta=\eps$, assim temos que: se $0<\abs{x-a}<\delta= \eps$ então

(5)
\[\abs{f(x)-L}=\abs{x-a}< \eps \]

Ou seja, $\abs{f(x)-L}=< \eps$.

Exercício. [Comportamentos Diferentes à Esquerda e à Direita] Seja $g=\dfrac{\abs{x}}{x}$ então $\limitex{0} g(x)$ não existe.

Resolucao Como:

(6)
\[g(x)= \begin{cases} 1 & \text{ se } x>0\\ -1& \text{ se } x<0 \end{cases} \]

Mostraremos que o limite não existe mostrando que não podemos fazer a distância entre $f(x)$ e um suposto limite $L$ menor que $\eps$, pois independente do quão próximo escolhermos o ponto da origem $\abs{x}<\delta$ teríamos :

(7)
\[\text{ se } x>0, \abs{f(x)-L}=\abs{1-L}<\eps \]
(8)
\[\text{ se } x<0, \abs{f(x)-L}=\abs{-1+L}<\eps \]

As equações anteriores teriam que ser satisfeitas simultaneamente para todo $\eps>0$.

Em especial, considerando o caso em que $\eps=1$ teríamos:

(9)
\[\text{ se } x>0,1-\eps <L<1+\eps \ssel 0<L<2 \]
(10)
\[\text{ se } x<0, -1-\eps <L<-1+\eps \ssel -2<L<0 \]

O que mostra que não existe $L$.

\psset{xunit=0.7cm,yunit=1.0cm,algebraic=true,dotstyle=o,dotsize=3pt 0,linewidth=0.8pt,arrowsize=3pt 2,arrowinset=0.25} \begin{pspicture*}(-3.21,-1.35)(3.24,1.51) \psaxes[labelFontSize=\scriptstyle,xAxis=true,yAxis=true,Dx=1,Dy=1,ticksize=-2pt 0,subticks=2]{->}(0,0)(-3.21,-1.35)(3.24,1.51) \psplot[linecolor=vermelho,plotpoints=200]{5.851733809755821E-6}{3.236003735086136}{1 } \psplot[linecolor=vermelho,plotpoints=200]{-3.2117099419109403}{-5.95979867241256E-7} {-1} \rput[bl](2,1.1){\color{vermelho}{$g$}} \psdots[linecolor=vermelho](0,-1) \psdots[dotstyle=*,linecolor=vermelho](0,1) \end{pspicture*}

Limites Laterais

No exemplo 3, vimos que a função $g$ definida como

(11)
\[g(x)=\left\{ \begin{tabular} [c]{l}% $1$ se $x\geq0$ $-1$ se $x<0$% \end{tabular} \right. \]

possui dois comportamentos distintos na vizinhança da origem. Se considerarmos valores maiores que $0$ teremos que $g(x)=1$ e logo

(12)
\[\lim_{x\to 0, x>0} g(x)=1, \]

enquanto que se consideramos valores menores que $0$ teremos que $g(x)=-1$ e logo

(13)
\[\lim_{x\to 0, x<0} g(x)=-1. \]

Indicaremos tais fatos por:

(14)
\[\lim_{x \to 0^+} g(x)=1, \qquad \lim_{x \to 0^-} g(x)=-1 \]

Definição Limite Lateral.

Seja $f$ uma função definida num intervalo aberto contendo $a$, exceto possivelmente em $a$ e seja $L$ um número real.

Dizemos que o limite lateral de $f(x)$ quando $x$ tende a $a$ pela esquerda é $L$

(15)
\[\limitex{a^-}f(x)=L \]

se para todo $\eps>0$ existe um $\delta=\delta(\eps)>0$ tal que

(16)
\[\text{se } a-\delta< x <a \text{ então } \abs{f(x)-L}<\eps. \]

De modo análogo, temos:

Definição 3.

Seja $f$ uma função definida num intervalo aberto contendo $a$, exceto possivelmente em $a$ e seja $L$ um número real.

Dizemos que o limite lateral de $f(x)$ quando $x$ tende a $a$ pela direita é $L$

(17)
\[\limitex{a^+}f(x)=L \]

se para todo $\eps>0$ existe um $\delta=\delta(\eps)>0$ tal que

(18)
\[\text{se } a< x <a+\delta \text{ então } \abs{f(x)-L}<\eps. \]

A diferença essencial da definição de limites laterais em relação a definição de limites é que nos limites laterais estamos considerando apenas valores menores que $a$ (ou seja intervalos da forma $ a-\delta< x <a$) nos limites pela esquerda e valores maiores que $a$ (ou seja intervalos da forma $ a< x <a+\delta$) nos limites pela direita.

A próxima proposição relaciona a existência dos limites laterais e do limite para uma função $f$.

Teorema 1.
Seja $f$ uma função definida num intervalo aberto contendo $a$, exceto possivelmente em $a$ e seja $L$ um número real. Então $\limitex{a} f(x)= L$ se e somente se $\limitex{a^+} f(x)= L$ e $\limitex{a^-}f(x)= L$.

O teorema anterior pode ser usado para demonstrar a existência ou não de alguns limites, como ilustrado nos exemplos seguintes:

Exercício. Mostre que $\limitex{0} \abs{x}=0$.

Resolução. Vamos demonstrar a existência do limite usando os limites laterais. Para tanto, começaremos calculando o limite pela direita. Como $\abs{x}=x$ se $x>0$, temos que

(19)
\[\limitex{0+} \abs{x}= \limitex{0+} x=0. \]

De maneira análoga, vamos calcular o limite pela esquerda. Como $\abs{x}=-x$ se $x<0$, temos que

(20)
\[\limitex{0-} \abs{x}=0. \]

Como ambos os limites laterais existem e são iguais temos pelo teorema 1 que:

(21)
\[\limitex{0} \abs{x} =0. \]

Exercício. Considere a função maior inteiro menor ou igual a $x$, i.e.,

\[\mint{x}=\max \{n \in \bbZ \,|\, n \leq x\}. \]

Para todo $n\in \bbN$, encontre

(22)
\[\limitex{n^+} \mint{x} \qquad \text{e} \quad \limitex{n^-} \mint{x} \]

Resolucao Começaremos calculando o limite $\limitex{n^+} \mint{x} $. Para isso seja $x$ tal que $x >n$. Como estamos interessados no comportamento numa vizinhança de $n$ podemos assumir sem perda de generalidade que $x<n+1$ e assim que $n<x<n+1$

Desta forma como para todo número real $x$, com $n \leq x < n+1$, tem-se que $\mint{x}=n$ e assim:

(23)
\[\limitex{n^+} \mint{x}=n \]

Para calcularmos o limite $\limitex{n^-} \mint{x} $, tomemos um $x$ satisfazendo $x <n$.

Como estamos interessados no comportamento numa vizinhança de $n$ podemos assumir sem perda de generalidade que $n-1<x$ e assim que $n-1<x<n$

(24)
\[\qquad \limitex{n^-} \mint{x}=n-1 \]

Como os limites laterais são distintos podemos concluir que não existe $\limitex{n} \mint{x}$ para todo $n\in \bbN$.

\psset{xunit=0.8cm,yunit=0.8cm,algebraic=true,dotstyle=*,dotsize=3pt 0,linewidth=0.8pt,arrowsize=3pt 2,arrowinset=0.25} \begin{pspicture*}(-3.44,-3.22)(5.58,3.74) \psaxes[labelFontSize=\scriptstyle,xAxis=true,yAxis=true,Dx=1,Dy=1,ticksize=-2pt 0,subticks=2]{->}(0,0)(-3.44,-3.22)(5.58,3.74) \psplot[linecolor=Maroon, linewidth=0.99pt,plotpoints=400]{-3.44}{-3.0001}{floor(x)} \psplot[linecolor=Maroon, linewidth=0.99pt,plotpoints=400]{-3}{-2.001}{floor(x)} \psplot[linecolor=Maroon, linewidth=0.99pt,plotpoints=400]{-2}{-1.001}{floor(x)} \psplot[linecolor=Maroon, linewidth=0.99pt,plotpoints=400]{-1}{-0.0001}{floor(x)} \psplot[linecolor=Maroon, linewidth=0.99pt,plotpoints=400]{0}{0.999}{floor(x)} \psplot[linecolor=Maroon, linewidth=0.99pt,plotpoints=400]{1}{1.999}{floor(x)} \psplot[linecolor=Maroon, linewidth=0.99pt,plotpoints=400]{2}{2.999}{floor(x)} \psplot[linecolor=Maroon, linewidth=0.99pt,plotpoints=400]{3}{3.999}{floor(x)} \psdots[dotstyle=o,linecolor=Maroon](-3,-4) \psdots[dotstyle=o,linecolor=Maroon](-2,-3) \psdots[dotstyle=o,linecolor=Maroon](-1,-2) \psdots[dotstyle=o,linecolor=Maroon](0,-1) \psdots[dotstyle=o,linecolor=Maroon](1,0) \psdots[dotstyle=o,linecolor=Maroon](2,1) \psdots[dotstyle=o,linecolor=Maroon](3,2) \psdots[dotstyle=o,linecolor=Maroon](4,3) \rput[tl](1.1181,-2.02){ Grafico de $\mint{x}$} \end{pspicture*}

Exercício. Considere a função

(25)
\[f(x) = \begin{cases} 3x-5 & \text{ se } x<2 \\ 2x-C & \text{ se } x \geq 2 \end{cases} \]

Determine o valor de $C$ de modo que o limite $\limitex{2}f(x)$ exista.

Resolução. Vamos começar calculando os limites laterais

(26)
\[\limitex{2^-}f(x)=\limitex{2^-}3x-5=1 \]
(27)
\[\limitex{2^+}f(x)=\limitex{2^-}2x-C=4-C \]


Pelo Teorema 1, para que o limite exista devemos ter:

(28)
\[\limitex{2^-}f(x)=\limitex{2^+}f(x) \]

E assim $1=4-C$, e logo $C=3$.

Propriedades do Limite de Funções

Teorema [Propriedades do Limite].

Seja $c\in \bbR$ e $f,g$ duas funções reais tais que
$\limitex{a} f(x)=A$ e $\limitex{a} g(x)= B$. Então:

  • $\limitex{a} (f(x)+g(x))=A+B$. (Limite da Soma)
  • $\limitex{a} (f(x)-g(x))=A-B$. (Lim Diferença)
  • $\limitex{a} (f(x)\cdot g(x))=AB$. (Limite do Produto)
  • $\limitex{a} (c f(x))=cA$. (Limite do Produto por Escalar)
  • Se $\limitex{a} g(x)=B\neq 0$ então $\limitex{a} \left(\dfrac{f(x)}{g(x)}\right)=\dfrac{A}{B}$. (Limite do Quociente)

Teorema [Propriedades do Limite 2].

  • $\limitex{a} \left| f(x)\right|=\left| A\right|$ (Limite do Módulo )
  • $\limitex{a}\left(f(x)^n\right) = A^n$ para todo $n\in \bbN$ (Lim de Potências)
  • $\limitex{a} \sqrt{f(x)} = \sqrt{A}$ (Limite da Raiz)

Exercício. Calcule $\limitex{2} x^3+3x+2$

Resolução.

(29)
\[ \begin{aligned} \limitex{2} x^3+3x+2&=&\limitex{2} x^3+\limitex{2}3x+\limitex{2}2\\ &=& \left( \limitex{2} x\right)^3+3\limitex{2}x+\limitex{2}2 \\ &=& 8+6+2=16 \end{aligned} \]

Exercício. Calcule $\limitex{a} \dfrac{x^4+2}{x^2+1}$

Resolução. Se $\limitex{a}x^2+1 \neq 0$ então

(30)
\[\begin{aligned} \limitex{a} \dfrac{x^4+2}{x^2+1}&=& \dfrac{\limitex{a}\left( x^4+2 \right)}{\limitex{a} \left( x^2+1 \right)} &=& \dfrac{\limitex{a} x^4\limitex{a}2 }{\limitex{a} x^2+ \limitex{a} 1 } &=& \dfrac{a^4+2}{a^2+1} \end{aligned} \]

De modo geral para um polinômio $p(x)$ podemos calcular o seu limite no ponto $a$ calculando simplesmente $p(a)$ ou seja por substituição direta de $x$ por $a$.

Teorema 4.
Dado um polinômio

\[p(x)=c_nx^n+c_{n-1}x^{n-1}+\cdots+c_1x+c_0 \]

então

\[\limitex{a} p(x)= p(a). \]

Demonstração. Vamos demonstrar por indução sobre o grau do polinômio. Se $p(x)$ é um polinômio de grau zero, ou seja constante, a igualdade é clara. Por hipótese indutiva, suponhamos que a igualdade anterior seja válida para os polinômios de grau menor igual que $n-1$. Agora usando a hipótese indutiva e [Propriedades do Limite] temos:

(31)
\[\begin{aligned} \scriptstyle \limitex{a} p(x)&=&\scriptstyle \left(\limitex{a}c_nx^{n-1}\right) \left(\limitex{a} x \right)+ \limitex{a} \left( c_{n-1}x^{n-1}+\cdots+c_1x+c_0\right)\\ &=&\scriptstyle c_na^{n-1}a+c_{n-1}a^{n-1}+\cdots+c_1a+c_0= p(a). \end{aligned} \]

Usando a propriedade [Propriedades do Limite] temos que para funções racionais também vale substituição direta para o cálculo de limites:

Teorema 5.
Dados polinômios $p(x)$ e $q(x)$ com $q(a)\neq0$ então

\[\limitex{a} \dfrac{p(x)}{q(x)}=\dfrac{p(a)}{q(a)}. \]

Exercício. Calcule $\limitex{2} \dfrac{x^3+12x+2}{4x^2+4x-2}$.

Resolução. Usando o exemplo anterior podemos calcular o limite por substituição e logo

(32)
\[\limitex{2} \dfrac{x^3+12x+2}{4x^2+4x-2}= \dfrac{8+24+2}{16+8-2}=\dfrac{34}{22} \]
  • Ressaltemos que nem todos os limites podem ser calculados por substituição direta.

  • Quando tivermos $\limitex{a} \dfrac{f(x)}{g(x)}$ com $\limitex{a} f(x) =0$ e $\limitex{a} g(x)=0$ dizemos que temos uma indeterminação do tipo $\dfrac{0}{0}$.

  • Nesses casos para o cálculo do limite temos que realizar uma simplificação antes da utilização das propriedades do limite.

  • Duas estratégias de simplificação usuais são a fatoração e a multiplicação pelo conjugado, como ilustram os exemplos a seguir.

Exercício.

Calcule $\limitex{2} \dfrac{x^2-6x+8}{x^2+x-6}$.

Resolução. Nesse caso não podemos realizar substituição direta nem tampouco usar a propriedade [Propriedades do Limite] pois o limite do denominador é $0$. Como o limite do numerador também é $0$ temos que $2$ é raiz de ambos os polinômios e assim:

\[\limitex{2} \dfrac{x^2-6x+8}{x^2+x-6} =\limitex{2}\dfrac{(x-2)(x-4)}{(x-2)(x+3)} \]

Agora para o cálculo do limite $x\neq 2$ e logo

\[\limitex{2} \dfrac{x^2-6x+8}{x^2+x-6} =\limitex{2}\dfrac{(x-2)(x-4)}{(x-2)(x+3)}=\limitex{2}\dfrac{x-4}{x+3}=-\dfrac{2 }{5}. \]

Agora retornaremos ao exemplo [#ex:limiteraiz]

Exercício. [Indeterminação do tipo 0/0]

Calcule $\limitex{0} \dfrac{\sqrt{x+25}-5}{x}$.

Resolução. Novamente não podemos realizar substituição direta nem tampouco usar a propriedade [Propriedades do Limite] pois o limite do denominador é $0$. Nesse caso multiplicaremos o numerador e o denominador pelo conjugado:

(33)
\[\begin{aligned} \limitex{0} \dfrac{\sqrt{x+25}-5}{x} &=& \limitex{0} \dfrac{(\sqrt{x+25}-5)(\sqrt{x+25}+5)}{x(\sqrt{x+25}+5)} &=& \limitex{0} \dfrac{x+25-25}{x(\sqrt{x+25}+5)} &=& \limitex{0} \dfrac{x}{x(\sqrt{x+25}+5)} &=& \limitex{0} \dfrac{1}{\sqrt{x+25}+5} \end{aligned} \]

E assim temos que:

(34)
\[\limitex{0} \dfrac{\sqrt{x+25}-5}{x}=\dfrac{1}{10} \]

Teorema 6.
[Teorema do Confronto] Dadas $f,g,h$ funções definidas num intervalo contendo o ponto $a$, exceto possivelmente em $a$, e tais que $f(x)\leq g(x)\leq h(x)$ nesse intervalo. Se $\limitex{a}f(x)=L=\limitex{a}h(x)$, então

\[\limitex{a}g(x)=L \]

Demonstração

Das hipóteses, temos que existe $\delta$ tal que $\left|g(x)-L\right|<\eps$ e $\left|h(x)-L\right|<\eps$ se $0<\left|x-c\right|<\delta$.

Podemos reescrever as desigualdades anteriores como

(35)
\[L-\eps<g(x)<L+\eps \]

e

(36)
\[L-\eps<h(x)<L+\eps \]

se $0<\left|x-c\right|<\delta$.

Logo

\[ \begin{aligned} -\eps<g(x)<f(x)<h(x)<L+\eps \text{ se } 0<\left|x-c\right|<\delta. \end{aligned} \]

equivalentemente

\[ \begin{aligned} -\eps<g(x)-L<f(x)-L<h(x)-L<\eps \text{ se } 0<\left|x-c\right|<\delta \end{aligned} \]

Consequentemente $\left|f(x)-L\right|<\max(\left|g(x)-L\right|,\left|h(x)-L\right|)<\eps$ se $0<\left|x-c\right|<\delta$.

Exercício. Mostre que $\limitex{0} x^2 \sen \dfrac{1}{x} =0$.

\newrgbcolor{ttwwff}{0.2 0.4 1} \newrgbcolor{qqttff}{0 0.2 1} \psset{xunit=16cm,yunit=24cm,algebraic=true,dotstyle=o,dotsize=3pt 0,linewidth=0.8pt,arrowsize=3pt 2,arrowinset=0.25} \scalebox{0.5}{ \begin{pspicture*}(-0.4,-0.12)(0.4,0.12) \psaxes[labelFontSize=\scriptstyle,xAxis=true,yAxis=true,labels=none,Dx=0.05, Dy=0.02,ticksize=0pt 0,subticks=2]{->}(0,0)(-0.4,-0.12)(0.4,0.12) \psplot[linecolor=red,plotpoints=100]{-0.4}{0.4}{x^2*SIN(1/x)} \psplot[linecolor=ttwwff,plotpoints=100]{-0.4}{0.4}{x^2} \psplot[linecolor=qqttff,plotpoints=100]{-0.4}{0.4}{-x^2} \rput[tl](0.17,0.073){$y=x^2$} \rput[tl](0.17,-0.07){$y=-x^2$} \rput[tl](0.17,0.03){$y=x^2\sen\frac{1}{x}$} \end{pspicture*}}

Resolução. Como

(37)
\[-1\leq \sen\dfrac{1}{x} \leq 1 \]

temos que

(38)
\[-x^2\leq x^2\sen\dfrac{1}{x} \leq x^2 \]

Como $\limitex{0} x^2=\limitex{0}-x^2=0$, pelo Teorema do Confronto temos que

(39)
\[\limitex{0} x^2 \sen \dfrac{1}{x} =0. \]

Teorema 7.
[Limite Fundamental]

(40)
\[\limitex{0}\dfrac{\sen(x)}{x}=1. \]

Demonstração Começaremos provando que para

\[ -\dfrac{\pi}{2}<x<\dfrac{\pi}{2} \]

valem as desigualdades:

\[0<\cos(x)<\dfrac{\sen x}{x}<\dfrac{1}{\cos(x)}. \]

Considere no círculo trigonométrico um ângulo $x$ com

\[0<x<\dfrac{\pi}{2}, \]

conforme apresentado na figura [#fig:desigualdadeseno], como os triângulos $\triangle OCB$ e $\triangle OAD$ são semelhantes, se denotarmos por $h$ o tamanho do segmento $AD$, por semelhança de triângulos temos que

\[\dfrac{h}{1}=\dfrac{\sen(x)}{\cos(x)} \]

e logo $\Area(\triangle OAD)=\dfrac{\sen(x)}{2\cos(x)}.$

Se denotarmos a área do setor circular delimitado pelos pontos $O,A,B$ por $\Area(OAB)$, pela figura ao lado é fácil ver que valem as desigualdades para $x<\dfrac{\pi}{2}$:

\[\Area(\triangle OBC)<\Area(OAB)<\Area(\triangle OAD) \]
\[\implica \dfrac{1}{2}\sen(x)\cos(x)<\dfrac{1}{2}x<\dfrac{\sen(x)}{2 \cos(x)}. \]

Dividindo por $\dfrac{\sen(x)}{2}$ temos:

\[ \cos(x) < \dfrac{x}{\sen(x)} < \dfrac{1}{\cos(x)}. \]

Finalmente, comparando os inversos dos três termos, obtemos:

\[\implica \cos(x)<\dfrac{\sen x}{x}<\dfrac{1}{\cos(x)} \]

O caso $-\dfrac{\pi}{2}<x<0$ é análogo e será deixado como exercício.

Assim como $\limitex{0} \cos(x)=1=\limitex{0}\dfrac{1}{\cos(x)} $ pelo Teorema do Confronto temos o limite desejado.

Exercício. Calcule $\limitex{0}\dfrac{1-\cos(x)}{x^2}$

Não podemos usar diretamente a regra do quociente pois $\limitex{0}{x^2}=0$. Para eliminar a indeterminação, multiplicaremos o numerador e o denominador por $1+\cos(x)$.

Resolução.

(41)
\[ \begin{aligned} \limitex{0}\dfrac{1-\cos(x)}{x^2}&=&\limitex{0}\dfrac{1-\cos(x)}{x^2}\dfrac{(1+\cos(x))}{(1+\cos(x))}\\ &=&\limitex{0}\dfrac{1-\cos^2(x)}{x^2}\dfrac{1}{(1+\cos(x))}\\ &=&\limitex{0}\dfrac{\sen^2(x)}{x^2}\dfrac{1}{1+\cos(x)}\\ &=&\limitex{0}\dfrac{\sen^2(x)}{x^2}\limitex{0}\dfrac{1}{1+\cos(x)}\\ &=& \dfrac{1}{2} \end{aligned} \]

Teorema 8.
[Mudança de Variáveis] Suponha que $\limitey{b}f(y)= L$. E suponha que $\Img g\subseteq \Dom f$, e que $\limitex{a} g(x)= b$ e que $g(x)\neq b$ numa vizinhança de $a$. Então

\[\limitex{a} f\circ g(x)= L \]

Demonstração. Seja $\epsilon >0$. Como $\limitey{b}f(y)= L$ existe $\delta >0$ tal que $0<|y-b|<\delta$ implica $|f(y)-L|<\epsilon$.

Como $\limitex{a} g(x)= b$, existe $\delta'>0$ tal que $0<|x-a|<\delta'$ implica $0<|g(x)-b|<\delta$. E logo $|f(g(x))- L|<\epsilon$ se $0<|x-a|<\delta'$.

Exercício. Mostre que $\limitex{2} \dfrac{\sen(x-2)}{x-2}=1$.

Resolução. Como $\limitex{0}\sen(x)=0$ como $\limitex{2} (x-2)=0$. Pelo Teorema 8 temos que: $\limitex{2} \dfrac{\sen(x-2)}{x-2}=\limitey{0} \dfrac{\sen(y)}{y}=1$.

Continuidade

De modo intuitivo, uma função $f:A\rightarrow B$, com $A,B\subset \bbR$ é dita contínua se variações suficientemente pequenas em $x$ resultam em variações pequenas de $f(x)$, ou equivalentemente, se para $x$ suficientemente próximo de $a$ tivermos que $f(x)$ é próximo de $f(a)$.

Antes de apresentarmos uma definição precisa de continuidade, vamos examinar alguns exemplos de comportamentos de continuidade e descontinuidades num ponto.

Começaremos por dois exemplos de descontinuidade:

\psset{xunit=0.7cm,yunit=0.7cm,algebraic=true,dotstyle=*,dotsize=4pt 0,linewidth=0.8pt,arrowsize=3pt 2,arrowinset=0.25} \begin{pspicture*}(-2.54,-2.34)(3.6,3.54) \psaxes[xAxis=true,yAxis=true,Dx=1,Dy=1,ticksize=-2pt 0,subticks=2]{->}(0,0)(-2.54,-2.34)(3.6,3.54) \psplot[plotpoints=100,linecolor=red]{-2.54}{3.5}{2*sin(x*1.57)}. \psdots[dotstyle=o](0.90,2) \psdots(0.90,3) \end{pspicture*}

Figura 4. Função descontínua em $x=1$.

No exemplo da figura 4 quando tomamos valores de $x$ diferentes de $1$ porém cada vez mais próximos de $1$, os valores de $f(x)$ se aproximam de $2$, porém o valor de $f(1)$ é $3$, e consequentemente temos uma descontinuidade nesse ponto.

No exemplo da figura 5 temos um tipo distinto de descontinuidade. Quando aproximamos de $1$ por valores maiores que $1$, temos que $f(x)$ se aproxima de $2$, enquanto que se aproximarmos de $1$ por valores menores que $1$ então $f(x)$ se aproxima de $1$. Veja que isso se manifesta no ‘'salto’' da função no ponto $x=1$.

\psset{xunit=0.8cm,yunit=0.7cm,algebraic=true,dotstyle=*,dotsize=4pt 0,linewidth=0.8pt,arrowsize=3pt 2,arrowinset=0.25} \begin{pspicture*}(-1.74,-1.1)(5,5) \psaxes[xAxis=true,yAxis=true,Dx=1,Dy=1,ticksize=-2pt 0,subticks=2]{->}(0,0)(-1.7,-1.1)(5,5) \psplot[plotpoints=200,linecolor=red]{-1.74}{1}{0.5*x+0.5} \psplot[plotpoints=200,linecolor=red]{1}{6}{0.25*x^2+1.75} \psdots[dotstyle=o](1,1) \psdots(1.04,2) \end{pspicture*}

Figura 5. Função descontínua em $x=1$

Definição 4.

Dada uma função $f:A\rightarrow B$ definida em pelo menos um conjunto aberto contendo o ponto $a$. Dizemos que a função $f(x)$ é contínua em $a$ se e somente se

(42)
\[\limitex{a}f(x)=f(a) \]

ou equivalentemente

(43)
\[\limitex{a^+}f(x)=\limitex{a^-}f(x)=f(a) \]

Uma função que é contínua em todo o seu domínio é dita contínua.

Teorema 9.
As seguintes funções são contínuas (em todo o seu domínio):

  • Funções Polinomiais.
  • Funções Racionais.
  • $\sen(x)$
  • $\cos(x)$
  • $c^x$

A demonstração da continuidade das funções polinomiais e racionais já foi feita implicitamente nos teoremas 45, nos quais provamos que dados polinômios $p(x)$ e $q(x)$ com $q(a)\neq 0$ então:

(44)
\[\limitex{a}p(x)=p(a) \qquad \limitex{a}\dfrac{p(x)}{q(x)}=\dfrac{p(a)}{q(a)} \]

Vamos provar que $\sen(x)$ é contínua. Para isso começamos mostrando que $ \abs{\sen(x)}< \abs{x}$. Considere no círculo trigonométrico um ângulo $x$ tal que

\[-\dfrac{\pi}{2}<x<\dfrac{\pi}{2}, \]

conforme apresentado na Figura [#fig:continuidadesen].

Geometricamente, temos que área do triângulo $OBC$, que vale $\abs{\sen(x)/2}$, é menor que a área do setor circular $OBC$, cujo valor é $\abs{\dfrac{x}{2}}$. Consequentemente para $-\dfrac{\pi}{2}<x<\dfrac{\pi}{2}$, vale a desigualdade:

\[\abs{\sen(x)}<\abs{x} \]

e assim

(45)
\[\begin{aligned} \left| {\sin x - \sin a} \right| &=& 2\left| {\sin \left( {\dfrac{{x - a}}{2}} \right)\cos \left( {\dfrac{{x + a}}{2}} \right)} \right| &=& 2\sin \left| {\dfrac{{x - a}}{2}} \right|\cos \left| {\dfrac{{x + a}}{2}} \right| \leq 2\left| \dfrac{{x - a}}{2} \right| &\leq& \left| {x - a} \right| \end{aligned} \]

E assim

(46)
\[ 0<\limitex{a} \left| {\sin x - \sin a} \right|< \limitex{a} \left| {x - a} \right| \]

Pelo Teorema do Confronto temos:

(47)
\[\limitex{a} \left| {\sin x - \sin a}\right|=0 \]

e logo $\limitex{a}\sin x = \sin a$. Consequentemente a função $\sen(x)$ é contínua.

Como consequência das propriedades do limite, temos as seguintes propriedades da continuidade de funções.

Teorema 10.
Se $f(x)$ e $g(x)$ são contínuas num ponto $a$, então:

  • $f(x)+g(x)$ é contínua em $a$
  • $f(x).g(x)$ é contínua em $a$
  • Se $g(a)\neq0$ então $f(x)/g(x)$ é contínua em $a$

Demonstração. Faremos apenas a demonstração do item a. A demonstração dos outros itens é similar e deixamos como exercício ao leitor.

Como as funções $f,g$ são contínuas em $a$ temos que os limites $\limitex{a}f(x)$ e $\limitex{a}g(x)$ existem e que:

(48)
\[\limitex{a}f(x)=f(a) \quad \limitex{a}g(x)=g(a) \]

Logo pelo limite da soma ([Propriedades do Limite]) temos que o limite da some existe e que:

(49)
\[\limitex{a}\left(f(x)+g(x)\right)=\limitex{a}f(x)+\limitex{a}g(x)=f(a)+g(a) \]

o que prova a continuidade da soma em $a$.

Como corolário do teorema anterior temos que a função $\tan(x)=\dfrac{\sen(x)}{\cos(x)}$ é contínua em todos os pontos do seu domínio, i.e, em $\bbR \backslash \{\dfrac{\pi}{2}+k\pi, \text{ com } k\in \bbZ\}$

\psset{xunit=0.5cm,yunit=0.5cm,algebraic=true,dotstyle=o,dotsize=3pt 0,linewidth=0.8pt,arrowsize=3pt 2,arrowinset=0.25} \begin{pspicture*}(-5.63,-5.69)(8.86,5.89) \psaxes[labelFontSize=\scriptstyle,xAxis=true,yAxis=true,Dx=1.57,Dy=2, ticksize=-2pt 0,subticks=2]{->}(0,0)(-5.63,-5.69)(8.86,5.89) \psplot[plotpoints=200]{-5.627030985000673}{8.861799147976855}{TAN(x)} \psline[linestyle=dashed,dash=3pt 3pt,linecolor=vermelho](-1.57,-5.69)(-1.57,5.89) \psline[linestyle=dashed,dash=3pt 3pt,linecolor=vermelho](1.57,-5.69)(1.57,5.89) \psline[linestyle=dashed,dash=3pt 3pt,linecolor=vermelho](7.85,-5.69)(7.85,5.89) \psline[linestyle=dashed,dash=3pt 3pt,linecolor=vermelho](4.71,-5.69)(4.71,5.89) \psline[linestyle=dashed,dash=3pt 3pt,linecolor=vermelho](-4.71,-5.69)(-4.71,5.89) \rput[bl](-6.6,-0.87){$f$} \end{pspicture*}

Figura 6. 

Podemos calcular o limite de funções compostas $\limitex{a}f\comp g (x)$, desde que a função $f$ seja contínua, calculando $f(\limitex{a}g(x))$.

Teorema [Limite da Composta].
Sejam $f$ e $g$ duas funções tais que $\Img f\subset \Dom g$. Se $f$ é contínua em $b$ e $\limitex{a}g(x)=b$ então $\limitex{a}{f(g(x))}=\limitey{b}f(y)=f(b)$.

Demonstração. Como $f$ é contínua em $b$, temos que $\limitex{b}f(x)=f(b)$. Por hipótese temos que $\limitex{a}g(x)=b$ Se $g(x)\neq b$ numa vizinhança de $a$, pelo Teorema 8

(50)
\[\limitex{a}f(g(x))=f(\limitex{a}g(x))=f(g(a)) \]

O outro caso é imediato.

Exercício. Mostre que $\limitex{0} \dfrac{\sen(x^2+4x+\pi)+2}{\cos(x^3+x^5)}=2$.

Resolução Como já dissemos as funções $\sen(x)$ e $\cos(x)$ são contínuas em todos os pontos.

Além disso temos:

(51)
\[\limitex{0}\left(x^2+4x+\pi\right) =\pi \quad \text{ e } \quad \limitex{0}x^3+x^5=0 \]

Logo,

(52)
\[\limitex{0}\sen(x^2+4x+\pi)+2=\sen(\limitex{0}x^2+4x+\pi)+2= \sen(\pi)+2=2 \]

e

(53)
\[\limitex{0}\cos(x^3+x^5)=\cos(\limitex{0}x^3+x^5)= \cos(0)=1 \]

Logo por [Propriedades do Limite] temos que:

(54)
\[\limitex{0} \dfrac{\sen(x^2+4x+\pi)+2}{\cos(x^3+x^5)}= \dfrac{\limitex{0}\left(\sen(x^2+4x+\pi)+2\right)}{\limitex{0}\cos(x^3+x^5)}=2 \]

Como consequência do Teorema do Limite da Composta temos que a composição de funções contínuas é contínuas:

Teorema 12.
Dadas funções $g:A\rightarrow B$ definida num aberto contendo o ponto $a$ e $f:B\rightarrow C$ definida num aberto contendo o ponto $g(a)$. Então se $g$ é contínua em $a$ e se $f$ é contínua em $g(a)$, então $f(g(x))$ é contínua em $a$.

Finalmente, temos que a inversa de uma função contínua é contínua.

Teorema 13.
Dado um intervalo $I$ e $f:I \to \bbR$ uma função contínua e monótona em $I$. Então $f^{-1} : f(I) \to \bbR$ é contínua em $f(I)$.

Como consequência do Teorema 13 temos que as funções trigonométricas inversas $\arcsen(x)$, $\arccos(x)$, $\arctan(x)$, etc. e a função $\log$ são contínuas em todos os pontos de seus respectivos domínios de definição.

E, ainda, como consequência do Teorema 12 temos que funções elementares, i.e, funções que são obtidas por soma, produto, quociente e compostas de funções polinomiais, racionais, trigonométricas, exponenciais e logarítmicas são contínuas em todos os pontos nos quais estão definidas.

Propriedades das Funções Contínuas

Teorema do Valor Intermediário

Geometricamente, o Teorema do Valor Intermediário nos diz que o gráfico de uma função contínua assume todos os valores entre $f(a)$ e $f(b)$, ou dito de outra forma, dado $d$ entre $f(a)$ e $f(b)$, o gráfico de $f(x)$ deve interceptar a reta horizontal $y=d$.

Teorema 14.
[Teorema do Valor Intermediário] Seja $f$ uma função contínua em todos os pontos de um intervalo fechado $\left[a,b \right]$ e com $f(a)\neq f(b)$ então para todo $d$ entre $f(a)$ e $f(b)$ existe $c \in (a,b)$ tal que $f(c)=d$;

tvi


Figura 7. 

Nessa seção apresentaremos algumas aplicações do Teorema do Valor Intermediário na demonstração de existência de soluções para equações. Para tanto, por sua utilidade, enunciaremos o Teorema do Valor Intermediário em uma forma especial e mais restrita: o Teorema de Bolzano.

Teorema [Teorema de Bolzano].
Seja $f$ uma função contínua em todos os pontos de um intervalo fechado $\left[a,b \right]$ e suponha que $f(a)$ e $f(b)$ tenham sinais opostos. Então existe um $c\in (a,b)$ tal que $f(c)=0$.

Demonstração.

O teorema é consequência da propriedade de completude dos números reais. Provaremos apenas o caso no qual $f (a) <d <f (b)$. A demonstração do outro caso, $f (b) <d <f (a)$, é similar.

Seja $S$ o conjunto de todos os $x$ em $[a, b]$ tais que $f(x) <d$. Então $S$ é um conjunto não-vazio pois $a$ é um elemento de $S$, e $S$ é limitado superiormente por $b$. Assim, por completude, existe o supremo $c = \sup S$. Provaremos que $f(c)=d$.

Dado $\eps> 0$, como $f$ é contínua, existe $\delta > 0$ tal que $ \abs{ f(x) - f(c)} < \eps$ sempre que $\abs{ x - c } <\delta$. Isso significa que

(55)
\[f(x) -\eps <f(c) <f (x) +\eps \]

para todo $x$ entre $c - \delta$ e $c + \delta$. Pelas propriedades do supremo, existem entre um $x^*$ entre $c-\delta$ e $c$ e que está contido em $S$, de modo que, para esse $x^*$

(56)
\[f (c) <f(x^*) +\eps <d +\eps. \]

Escolha $\hat{x}$ entre $c$ e $c + \delta$, que obviamente não estará contido em $S$, e dessa forma teremos:

(57)
\[f (c)> f (\hat{x}) -\eps \geq d -\eps. \]

Combinando as desigualdades anteriores temos que

(58)
\[ d -\eps <f (c) <d +\eps \]

para todo $\eps> 0$, e assim temos que $f(c) = d$.

O teorema anterior nos diz que o gráfico de uma função contínua que em $a$ está abaixo do eixo $x$ e em $b$ está sobre este (ou vice-versa), em algum ponto do intervalo $[a,b]$ deve cruzar o eixo $x$.

bolzano

Exercício. Mostre que a equação $\cos(x)=x$ tem pelo menos uma solução no intervalo $\left[0,\pi\right]$.

Resolução. Note que a equação anterior é equivalente $\cos(x)-x=0$. Assim começaremos considerando a função $g(x)=cos(x)-x$, que é contínua pois é soma de funções contínuas.

Agora observamos que $g(0)=\cos(0)-0=1$, e logo $g(0)>0$ e que $g(\pi)=\cos(\pi)-\pi=-1-\pi$, e logo $g(\pi)<0$.

Logo pelo Teorema de Bolzano existe $c\in (0,\pi)$ tal que $g(c)=\cos(c)-c=0$, e desta forma temos que a equação tem uma solução.

\newrgbcolor{qqzzff}{0 0.6 1} \psset{xunit=1.3cm,yunit=1.3cm,algebraic=true,dotstyle=o,dotsize=3pt 0,linewidth=0.8pt,arrowsize=3pt 2,arrowinset=0.25} \begin{pspicture*}(-1.76,-1.32)(3.14,2.08) \psaxes[labelFontSize=\scriptstyle,xAxis=true,yAxis=true,Dx=1,Dy=1,ticksize=-2pt 0,subticks=2]{->}(0,0)(-1.76,-1.32)(3.14,2.08) \psplot[plotpoints=200,linecolor=vermelho]{-1.76}{3.14}{COS(x)} \psplot[linecolor=qqzzff,plotpoints=200]{-1.76}{3.14}{x} \rput[tl](0.6,-0.54){$y=\cos(x)$} \rput[tl](0.54,1.48){$y=x$} \psdots[dotstyle=*,linecolor=darkgray](0.74,0.74) \end{pspicture*}

Figura 8. Intersecção dos gráficos de $y=x$ e $y=\cos(x)$
\colorlet{xdxdff}{azul} \psset{xunit=1.2cm,yunit=0.4cm,algebraic=true,dotstyle=o,dotsize=3pt 0,linewidth=0.8pt,arrowsize=3pt 2,arrowinset=0.25} \begin{pspicture*}(-1.83,-6.89)(4.18,3.43) \psaxes[labelFontSize=\scriptstyle,xAxis=true,yAxis=true,Dx=1.57,Dy=2, ticksize=-2pt 0,subticks=2]{->}(0,0)(-1.83,-6.89)(4.18,3.43) \psplot[plotpoints=200,linecolor=vermelho]{-1.828000813535533}{4.178443858115672}{COS(x)-x} \rput[tl](0.42,-3.11){$y=\cos(x)-x$} \psline[linestyle=dashed,dash=5pt 5pt](3,14,0)(3,14,-4.14) \psdots[dotstyle=*,linecolor=xdxdff](0,1) \psdots[dotstyle=*,linecolor=xdxdff](3.14,-4.14) \rput[bl](2.78,-2.04){$c$} \end{pspicture*}

Figura 9. Gráfico de $y=\cos(x)-x$.

Exercício. Mostre que a equação $3^x=x^2+4$ tem pelo menos uma solução no intervalo $\left(1,2\right)$.

Resolução. Note que a equação anterior é equivalente $3^x - x^2 - 4=0$. Assim começaremos considerando a função $g(x)=3^x - x^2 - 4$, que é contínua pois é soma de funções contínuas.

Agora observamos que $g(0)=3^0 - 4=-3$, e logo $g(0)<0$ e que $g(2)=9-4-4=1$, e logo $g(2)>0$.

Logo pelo Teorema de Bolzano existe $c\in (1,2)$ tal que $f(c)=3^c-c^2-4=0$, e desta forma temos que a equação tem pelo menos uma solução.

\psset{xunit=1.9324096372070039cm,yunit=0.3374048572901118cm,algebraic=true,dotstyle=o,dotsize=3pt 0,linewidth=0.8pt,arrowsize=3pt 2,arrowinset=0.25} \begin{pspicture*}(-0.69,-6.05)(2.42,5.81) \psaxes[labelFontSize=\scriptstyle,xAxis=true,yAxis=true,Dx=1,Dy=5,ticksize=-2pt 0,subticks=2]{->}(0,0)(-0.69,-6.05)(2.42,5.81) \psplot[plotpoints=200,linecolor=azul]{-0.6864372551954321}{2.418494475880005}{3^x-x^2-4} \rput[bl](0.3,-4.39){$3^x-x^2-4$} \end{pspicture*}

Figura 10. Gráfico de $y=3^x-x^2-4$.

Teorema 16.
Uma função contínua $f: I \to \bbR$ de um intervalo fechado $I=[a, b]$ em $\bbR$ é injetiva se e somente se a função $f$ é estritamente monotônica em $[a, b]$.

Demonstração. Se $f$ é estritamente crescente ou decrescente em qualquer conjunto $I$, a aplicação $f: I \to \bbR$ é obviamente injetiva.

Assim, a parte mais substancial da proposição consiste na afirmação que cada função injetiva e contínua $f: [a, b] \to \bbR$ é uma função monótona.

Vamos provar por absurdo, suponha que existam três pontos $x_1 <x_2 <x_3$ em $[a, b]$, tal que $f(x_2)$ não se encontra entre $f(x_1)$ e $f(x_3)$. Sem perda de generalidade vamos assumir que $f(x_1)$ está entre $f(x_2)$ e $f(x_3)$ . Por hipótese $f$ é contínua em $[x_2, x_3]$. Portanto, pelo Teorema do Valor Intermediário, existe $x'$ neste intervalo tal que $f(x') = f (x_1)$. Temos, então, $x_1 <x'$, mas $f(x_1) = f (x')$, que é incompatível com a injetividade da função.

Valores Extremos

Teorema 17.
Se uma função $ f $ é contínua em um intervalo fechado $[ a, b ]$, então ela é limitada nesse intervalo.

Demonstração Suponha que $ f $ não é limitada no intervalo $[ a, b ]$. Deixe $ c $ ser o ponto médio de $[ a, b ]$. Então $ f $ será ilimitada em pelo menos um dos dois intervalos de $[ a, c ]$ e $[ c, b]$ . Nós escolhemos o intervalo em que é ilimitada (no caso, em que a função seja ilimitada em ambos os intervalos, nós escolheremos o intervalo de esquerda). Denotaremos esse intervalo como $[ a_1, b_1 ]$.

Este processo de bisseção será realizado indefinidamente e o intervalo $[ a_ {n + 1}, b_ {n + 1} ]$ indicará a metade de $[ a_n, b_n ]$ em que $ f $ é ilimitada. Caso seja ilimitada em ambas as metades, a metade esquerda será selecionada.

O comprimento do $n$-ésimo intervalo é $(b-a)/2^n $.

Deixe $ A $ denotar o conjunto de pontos de extremidade mais à esquerda $ a, a_1, a_2, a_3 ... $ assim obtido. Deixe $ \alpha $ denotar o supremo $ A $. Então $ \alpha $ encontra-se em $ [a, b] $.

Como $f$ é contínua em $\alpha $, existe um $ \delta> 0 $ tal que

(59)
\[| f (x) | <1 + | f (\alpha)| \]

no intervalo de $ (\alpha - \delta, \alpha + \delta) $ (No caso $ \alpha = a $, o intervalo deve ser $ [a, a + \delta) $. Em caso $ \alpha = b $, o intervalo deve ser $ (b -\delta, b] $)

No entanto, o intervalo [$ a_n, b_n $] situa-se dentro do intervalo de $ (\alpha - \delta, \alpha + \delta) $, pois $ (b-a) / 2^n <\delta $.

Portanto, $f$ é limitada em $ (b-a) / 2^n $, o que é uma contradição.

Definição 5.
Seja $I$ um intervalo e $f:I\to \bbR$ uma função.

  • Diremos que $x_0\in I$ é um ponto de máximo global (ou absoluto) de $f$, se $f(x)\leq f(x_0)$, para todo $x\in I$. Neste caso, diremos que $f(x_0)$ é máximo global.

  • Diremos que $x_0\in I$ é um ponto de mínimo global de $f$, se $f(x)\geq f(x_0)$, para todo $x\in I$. Neste caso, diremos que $f(x_0)$ é mínimo global.

  • Um ponto $x_0\in I$ será dito um ponto extremo global, se $x_0$ for um ponto de máximo global ou um ponto de mínimo global.

Teorema 18.
[Teorema de Weierstrass do Valor Extremo] Seja $f$ uma função contínua em um intervalo $[a, b]$, então $ f $ atinge seus valores máximos e mínimos em $[a, b]$.

Demonstração. Como $ f $ é contínua, então $ f $ possui a menor cota superior, que denominaremos $ M $. Suponha que não há nenhum valor $ c \ in [a, b] $ para que $ f (c) = M $. Portanto, $ f (x) <M $ para todo $ x \in [a, b] $. Defina uma nova função $ g $ por

$ g (x) = \dfrac {1} {M-f (x)} $

Observe que $ g (x)> 0 $ para cada $ x \in [a, b]$ e que $ g $ é contínua e limitada em $[a, b]$. Portanto, existe $ K> 0 $ tal que $ g (x) \leq K $ para cada x  in [a, b] $ $. Uma vez que para cada $ x \ in [a, b] $,

(60)
\[ g (x) = \dfrac {1} {M-f (x)} \leq K \text{ é equivalente a } f (x) \leq M- \dfrac {1} {K} \]

Contradizemos o fato de que $ M $ foi assumido como sendo o extremo superior de $ f $ em [a, b]. Assim, deve haver uma valor $ c \in [a, b]$ tal que $ F (C) = M $.

Limites Infinitos e no Infinito

Limites no Infinito

Vamos considerar a função $f(x)= \dfrac{x}{x^2+1}$, cujo gráfico é apresentado na Figura 11.

\scalebox{0.6}{ \psset{xunit=0.49cm,yunit=1.6cm,algebraic=true,dimen=middle,dotstyle=o,dotsize=3pt 0,linewidth=0.8pt,arrowsize=3pt 2,arrowinset=0.25} \begin{pspicture*}(-10.5,-1.2)(10.5,1.3) \psaxes[labelFontSize=\scriptstyle,xAxis=true,yAxis=true,Dx=2.,Dy=0.5,ticksize=-2pt 0,subticks=2]{->}(0,0)(-12.,-1.2)(12.,1.3) \psplot[linewidth=1.2pt,linecolor=vermelho,plotpoints=200]{-12.0}{12.0}{x/(x^(2.0)+1.0)} \rput[tl](2.87,0.814){$\dfrac{x}{x^2+1}$} \rput[tl](7.6,-0.495){$x \to \infty$} \rput[tl](-10,0.464){$x \to -\infty$} \begin{scriptsize} \end{scriptsize} \end{pspicture*}}

Figura 11. Gráfico de $f(x)= \dfrac{x }{x^2+1}$

Podemos observar que conforme os valores de $x$ se tornam suficientemente grandes temos que os valores da função se aproximam de $0$. Denotaremos tal fato por

(61)
\[\limitex{\infty} \dfrac{x}{x^2+1}=0 \]

Por outro lado, conforme os valores de $x$ se tornam suficientemente grandes negativos (negativos e com valores absolutos grandes) temos que os valores da função também se aproximam de 0. Denotaremos tal fato por

(62)
\[\limitex{-\infty} \dfrac{x}{x^2+1}=0 \]

Podemos modificar a noção de limite anterior de modo a lidar com esses casos. A modificação essencial é formalizar a afirmação que ‘'se $x$ é suficientemente grande’' através de ‘'existe $\delta$ tal que se $x>\delta$’'.

Definição Limite no Infinito.
Seja $f$ uma função definida para $x>c$ para algum $c\in \bbR$ e seja $L$ um número real. Dizemos que

(63)
\[\limitex{\infty}f(x)=L \]

se para todo $\eps>0$ existe um $\delta>0$ tal que

(64)
\[\text{se } x> \delta \text{ então } \abs{f(x)-L}<\eps. \]

Seja $f$ uma função definida para $x<c$ para algum $c\in \bbR$ e seja $L$ um número real. Dizemos que

(65)
\[\limitex{-\infty}f(x)=L \]

se para todo $\eps>0$ existe um $\delta>0$ tal que

(66)
\[\text{se } x< \delta \text{ então } \abs{f(x)-L}<\eps. \]

Exercício. Mostre a partir da definição que $\limitex{\infty} \dfrac{1}{x}=0$.

Resolução. Queremos mostrar que existe $\delta$ tal que se $x>\delta$ então $\abs{f(x)}<\eps$.

Para tanto começaremos determinando quando $\abs{f(x)}<\eps$. Como estamos interessados no comportamento no infinito, podemos supor sem perda de generalidade que $x>0$, e assim temos que a desigualdade $\nicefrac{1}{x}<\eps$ é equivalente a $x>\nicefrac{1}{\eps}$. Assim escolhemos $\delta=\nicefrac{1}{\eps}$. Quando $x>\delta$ então $x>\nicefrac{1}{\eps}$ e assim $0<\nicefrac{1}{x}<\eps$. O que prova que $\limitex{\infty} \dfrac{1}{x}=0$.

Exercício. Mostre a partir da definição que $\limitex{\infty} \dfrac{1}{\sqrt{x}}=0$.

Resolução. Queremos mostrar que existe $\delta$ tal que se $x>\delta$ então $\abs{f(x)}<\eps$.

Para tanto começaremos determinando quando $\abs{f(x)}<\eps$. Como estamos interessados no comportamento no infinito, podemos supor sem perda de generalidade que $x>0$, e assim temos que a desigualdade $\nicefrac{1}{\sqrt{x}}<\eps$ é equivalente a $x>\nicefrac{1}{\eps^2}$. Assim escolhemos $\delta=\nicefrac{1}{\eps^2 }$. Quando $x>\delta$ então $x>\nicefrac{1}{\eps^2}$ e assim $0<\nicefrac{1}{\sqrt{x}}<\eps$. O que prova que $\limitex{\infty} \dfrac{1}{\sqrt{x}}=0$.

Limites Infinitos

No Exercício Resolvido [#ex:umsobreabsx] vimos que não existe o limite $\limitex{0}\dfrac{1}{\abs{x}}$.

\scalebox{0.7}{ \newrgbcolor{yqqqqq}{0.5019607843137255 0. 0.} \psset{xunit=0.8cm,yunit=0.9cm,algebraic=true,dimen=middle,dotstyle=o,dotsize=3pt 0,linewidth=0.8pt,arrowsize=3pt 2,arrowinset=0.25} \begin{pspicture*}(-3.363919646746774,-0.5053385717017645)(3.4181941906499764,4.683739891107842) \psaxes[labelFontSize=\scriptstyle,xAxis=true,yAxis=true,Dx=1.,Dy=1.,ticksize=-2pt 0,subticks=2]{->}(0,0)(-3.363919646746774,-0.5053385717017645)(3.4181941906499764,4.683739891107842) \psplot[linewidth=1.2pt,linecolor=yqqqqq,plotpoints=100]{-3.363919646746774}{3.4181941906499764}{1.0/abs(x)} \begin{scriptsize} \end{scriptsize} \end{pspicture*}}

Figura 12.  Não existe $\limitex{0 }\dfrac{1}{\abs{x}}$

~ End Slide

Em especial, vimos que escolhendo o valor de $x$ suficientemente pequeno podemos fazer o valor da função $\dfrac{1}{\abs{x}}$ arbitrariamente grande. Nesses casos nos quais o limite não existe, mas a função toma valores que crescem de forma ilimita dizemos que o limite da função é infinito.

Vejamos outro exemplo:

Os limites $\limitex{4^+}\dfrac{7}{x-4}$ e $\limitex{4^-}\dfrac{7}{x-4}$.

\newrgbcolor{yqqqqq}{0.5019607843137255 0. 0.} \newrgbcolor{qqzzff}{0. 0.6 1.} \psset{xunit=0.13cm,yunit=0.13cm,algebraic=true,dimen=middle,dotstyle=o,dotsize=3pt 0,linewidth=0.8pt,arrowsize=3pt 2,arrowinset=0.25} \begin{pspicture*}(-20.,-18.)(20.,18.) \psaxes[labelFontSize=\scriptstyle,xAxis=true,yAxis=true,Dx=5.,Dy=5.,ticksize=-2pt 0,subticks=2]{->}(0,0)(-20.,-18.)(20.,18.) \psplot[linewidth=1.2pt,linecolor=yqqqqq,plotpoints=200]{-20.0}{-0.1}{7.0/(x-4.0)} \psplot[linewidth=1.2pt,linecolor=yqqqqq,plotpoints=200]{0.10}{20.0}{7.0/(x-4.0)} \psline[linestyle=dotted,linecolor=qqzzff](4.,-18.)(4.,18.) \rput[tl](5.048816009657766,-5.416255739016001){$x=4$} \rput[tl](6.557713871280784,14.294033761411914){$\dfrac{7}{x-4}$} \begin{scriptsize} \end{scriptsize} \end{pspicture*}

Figura 13. 

A partir da Figura [#graf:limiteinf] podemos observar que quando $x$ tende a $4$ pela direita, isto é, por valores maiores que $4$ a função $\dfrac{7}{x-4}$ cresce indefinidamente, tomando valores arbitrariamente grandes. Enquanto que quando $x$ tende a $4$ pela esquerda, isto é, por valores menores que $4$ a função $\dfrac{7}{x-4}$ decresce indefinidamente, tomando valores arbitrariamente grandes e negativos.

Representamos esses comportamentos por:

(67)
\[\limitex{4^+}\dfrac{7}{x-4}=\infty \quad \text{ e }\quad \limitex{4^-}\dfrac{7}{x-4}=-\infty \]

Definição Limites Infinitos.

Seja $f$ uma função definida num intervalo aberto contendo $a$, exceto possivelmente em $a$.

  • Dizemos que $\limitex{a}f(x)=\infty$ se para todo $\eps>0$ existe um $\delta>0$ tal que
(68)
\[\text{se } 0<\abs{x-a}<\delta \text{ então } f(x)>\eps. \]
  • Dizemos que $\limitex{a}f(x)=-\infty$ se para todo $\eps>0$ existe um $\delta>0$ tal que
(69)
\[\text{se } 0<\abs{x-a}<\delta \text{ então } f(x)<\eps. \]

Definição Limites Infinitos 2.

  • Dizemos que $\limitex{a^+}f(x)=\infty$ se para todo $\eps>0$ existe um $\delta>0$ tal que
(70)
\[\text{se } a<x<a+\delta \text{ então } f(x)>\eps. \]
  • Dizemos que $\limitex{a^-}f(x)=\infty$ se para todo $\eps>0$ existe um $\delta>0$ tal que
(71)
\[\text{se } a-\delta<x<a \text{ então } f(x)>\eps. \]

De maneira análoga, podemos definir os limites laterais infinitos negativos : $\limitex{a^+}f(x)=- \infty$ e $\limitex{a^-}f(x)=-\infty$ e os limites infinitos no infinito $\limitex{\infty}f(x)= \infty$, $\limitex{\infty}f(x)=-\infty$, $\limitex{-\infty}f(x)= \infty$ e $\limitex{-\infty}f(x)= -\infty$.

Exercício. Mostre que $\limitex{\infty}x=\infty$.

Resolução. Pela definição temos que mostrar que dado $\eps>0$ existe $\delta>0$ tal que se $x>\delta$ então $f(x)>\eps$. A demonstração nesse caso é imediata pois escolhendo $\delta=\eps$ temos o resultado desejado.

Exercício. Mostre que $\limitex{\infty}x^2=\infty$.

Resolução. Nesse caso basta escolher $\delta=\sqrt{\eps}$ para termos que se $x>\delta>0$ então $x^2>\eps$.

Teorema 19.

  • Se $f(x)>g(x)$ e $\limitex{a} g(x)=\infty$ então $\limitex{a} f(x)=\infty$.

  • Se $f(x)<g(x)$ e $\limitex{a} g(x)=-\infty$ então $\limitex{a} f(x)=-\infty$.

  • Se $f(x)>0$ e $\limitex{a} f(x)=0$ então $\limitex{a} \dfrac{1}{f(x)}=\infty$.

  • Se $f(x)<0$ e $\limitex{a} f(x)=0$ então $\limitex{a} \dfrac{1}{f(x)}=-\infty$.

  • Se $f(x)\neq 0$ $\limitex{a} f(x)=\infty$ ou $\limitex{a} f(x)=-\infty$ então $\limitex{a} \dfrac{1}{f(x)}=0$.

Exemplos Como corolário do teorema anterior, temos os seguintes limites, que são facilmente obtidos através de comparação com uma das funções $x$ e ou $-x$.

  • Dado $c>0$ então $\limitex{\infty} c^x = \infty$.
  • Dado $k\in \bbN^*$ então $\limitex{\infty} x^k=\infty$.
  • Dado $k\in \bbN^*$ ímpar então $\limitex{-\infty} x^k=-\infty$.
  • Dado $k\in \bbN^*$ par então $\limitex{-\infty} x^k=\infty$.
  • $\limitex{\infty} \ln x=\infty$

Propriedades do Limite Infinito e no Infinito

Teorema [Propriedades Aditivas do Limite Infinito].

Sejam $f(x) ,g(x), h(x) \text{ e } m(x) $ funções, tais que:

(72)
\[\limitex{a} f(x)=\infty,\quad\qquad\limitex{a} g(x)=\infty \]
(73)
\[\limitex{a} h(x)=-\infty\quad\qquad\limitex{a} m(x)=-\infty \]

e seja $n(x)$ uma função limitada. Então:

  • $\limitex{a} (f(x)+g(x))=\infty$.

  • $\limitex{a} (f(x)-h(x))=\infty$.

  • $\limitex{a}(f(x)+n(x))=\infty$.

  • $\limitex{a}(h(x)+n(x))=-\infty$.

Teorema Continuação.

  • $\limitex{a} ( h(x)+m(x))=-\infty$.

  • $\limitex{a} (h(x)-f(x))=-\infty$.

Teorema [Propriedades Multiplicativas do Limite Infinito].
Seja $c$ um número real e $f(x),g(x),h(x),m(x),n(x)$ e $p(x)$ funções , tais que

(74)
\[\limitex{a} f(x)=\infty,\quad\qquad\limitex{a} g(x)=\infty \]
(75)
\[\limitex{a} h(x)=-\infty\quad\qquad\limitex{a} m(x)=-\infty \]
(76)
\[\limitex{a} n(x)=L_{1}>0\quad\qquad\limitex{a} p(x)=L_{2}<0 \]

Então:

  • $\limitex{a} n(x)f(x)=\infty$

  • $\limitex{a} p(x)f(x)=-\infty$

  • $\limitex{a} n(x)h(x)=-\infty$

Teorema [Continuação.].

  • $\limitex{a} p(x)h(x)=\infty$

  • $\limitex{a} f(x)\cdot g(x)=\infty$

  • $\limitex{a} f(x)\cdot h(x)=-\infty$

  • $\limitex{a} h(x)\cdot m(x)=\infty$

As propriedades anteriores permanecem válidas se trocamos o limite no ponto $a$ por limites laterais ou por limites infinitos.

Teorema  [Propriedades do Limite no Infinito] .

Seja $c$ um número real e $f,g$ duas funções reais tais que $\limitex{\infty} f(x)=A$ e $\limitex{\infty} g(x)= B$. Então:

  • $\limitex{\infty} (f(x)+g(x))=A+B$.
  • $\limitex{\infty} (f(x)-g(x))=A-B$.
  • $\limitex{\infty} (f(x)\cdot g(x))=AB$.
  • $\limitex{\infty} (c f(x))=cA$.

Teorema [Continuação].

  • Se $B\neq 0$ então $\limitex{\infty} \left(\dfrac{f(x)}{ g(x)}\right)=\dfrac{A}{B}$.
  • $\limitex{\infty} \left| f(x)\right|=\left| A\right|$.
  • $\limitex{\infty}\left(f(x)^n\right) = A^n$
  • $\limitex{\infty} \sqrt{f(x)} = \sqrt{A}$

Indeterminações

Quando tivermos $\limitex{a} \dfrac{f(x)}{g(x)}$ com $\limitex{a} f(x) =\infty$ e $\limitex{a} g(x)=\infty$ dizemos que temos uma indeterminação do tipo $\dfrac{\infty}{\infty}.$

Nesses casos para o cálculo do limite, de modo análogo as indeterminações do tipo $\dfrac{0}{0}$, temos que realizar uma simplificação antes da utilização das propriedades do limite.

As estratégias de simplificação usuais são a fatoração e a multiplicação pelo conjugado e também multiplicar ou dividir o numerador e o denominador por um termo apropriado, como ilustram os exemplos a seguir.

Exercício. Calcule $\limitex{\infty} \dfrac{x^2+1}{x^2-1}$.

Resolução.

(77)
\[\begin{aligned} \limitex{\infty} \dfrac{x^2+1}{x^2-1}&=& \limitex{\infty} \dfrac{x^2+1 }{x^2-1 }\dfrac{\div x^2}{\div x^2} &=& \limitex{\infty} \dfrac{1+\nicefrac{1}{x^2} }{1-\nicefrac{1}{x^2}} \end{aligned} \]

Como $\limitex{\infty}\nicefrac{1}{x^2}=\limitex{\infty}\nicefrac{1}{x} \limitex{\infty}\nicefrac{1}{x}=0$, temos que $\limitex{\infty}1+\nicefrac{1}{x^2}=1= \limitex{\infty} 1-\nicefrac{1}{x^2}$

Temos que

(78)
\[\limitex{\infty} \dfrac{x^2+1}{x^2-1}=1 \]

Exercício. Calcule $\limitex{\infty}(2x^3-3x^2+1)$.

Resolução. Colocando o termo de maior grau em evidência:

(79)
\[ \begin{aligned} \limitex{\infty}(2x^3-3x^2+1)&=& x^3\limitex{\infty}2-3\nicefrac{1}{x}+\nicefrac{1}{x^3} &=& \infty \cdot 2=\infty \end{aligned} \]

Exercício. Calcule $\limitex{\infty} \dfrac{2x^3+3x^2+1}{4x^2-2x+1}$.

Resolução.

(80)
\[ \begin{aligned} \limitex{\infty}\dfrac{2x^3+3x^2+1}{4x^2-2x+1}&=& \dfrac{x^3(2+3\nicefrac{1}{x}+\nicefrac{1}{x^3}}{x^2(4-2\nicefrac{1}{x}+\nicefrac{1}{x^2}}\\ &=& x \dfrac{(2+3\nicefrac{1}{x}+\nicefrac{1}{x^3}}{(4-2\nicefrac{1}{x}+\nicefrac{1}{x^2}}\\ &=& \infty \cdot \dfrac{2}{4}=\infty \end{aligned} \]

Exercício. Mostre que $\limitex{\infty} \dfrac{x}{\sqrt{9x^2+1}}=\dfrac{1}{3}$.

Resolução.

(81)
\[\begin{aligned} \limitex{\infty} \dfrac{x}{\sqrt{9x^2+1}}&=& \limitex{\infty} \dfrac{x }{\sqrt{9x^2+1}}\dfrac{\div x}{\div x} &=& \limitex{\infty} \dfrac{1 }{\sqrt{9 + \nicefrac{1}{x^2}}} \end{aligned} \]

Como $\limitex{\infty}\sqrt{9 + \nicefrac{1}{x^2}}=\sqrt{\limitex{\infty} 9 + \nicefrac{1}{x^2}}=3$ então

(82)
\[\limitex{\infty} \dfrac{x}{\sqrt{9x^2+1}}=\dfrac{1}{3}. \]

Exercício. Calcule $\limitex{\infty} \dfrac{5x^3+x^2-3}{2x^3-x+5}$.

Resolução.

(83)
\[\begin{aligned} \limitex{\infty} \dfrac{5x^3+x^2-3}{2x^3-x+5}&= & \limitex{\infty} \dfrac{5x^3+x^2-3}{2x^3-x+5} \dfrac{\div x^3}{\div x^3}\\ &= & \limitex{\infty} \dfrac{5+\nicefrac{1}{x}-3\nicefrac{1}{x^3}}{2-\nicefrac{1}{x^2}+5\nicefrac{1}{x^3}} \\ &=& \dfrac{5}{2} \end{aligned} \]

Exercício. Calcule $\limitex{\infty} \dfrac{5x^2+x-3}{4x^4-x+2}$.

Resolução.

(84)
\[\begin{aligned} \limitex{\infty} \dfrac{5x^2+x-3}{4x^4-x+2}&=& \limitex{\infty} \dfrac{5x^2+x-3}{4x^4-x+2}\dfrac{\div x^4}{\div x^4} &=& \limitex{\infty} \dfrac{5\nicefrac{1}{x^2}+\nicefrac{1}{x^3}-3\nicefrac{1}{x^4}}{4-\nicefrac{1}{x^3}+2\nicefrac{1}{x^4}} &=&0 \end{aligned} \]

O Número $e$ e as Funções Exponencial e Logaritmo

O próximo limite é conhecido como Limite Exponencial Fundamental é a base dos logaritmos naturais ou neperianos.

Teorema 26.
[Segundo Limite Fundamental]

(85)
\[\limitex{\infty} \left( 1+\dfrac{1}{x} \right)^x=e, \]

onde $e \approx 2,71828$ é a constante de Euler.

\psset{xunit=1.0cm,yunit=1.0cm,algebraic=true,dimen=middle,dotstyle=o,dotsize=5pt 0,linewidth=0.8pt,arrowsize=3pt 2,arrowinset=0.25} \begin{pspicture*}(-0.7,-0.74)(6.5,4.2) \psaxes[labelFontSize=\scriptstyle,xAxis=true,yAxis=true,Dx=1.,Dy=1.,ticksize=-2pt 0,subticks=2]{->}(0,0)(-0.7,-0.74)(6.5,4.2) \psplot[linestyle=dashed,dash=1pt 1pt,linecolor=azul]{-0.7}{6.5}{(--2.718281828459045-0.*x)/1.} \psplot[linewidth=1.2pt,linecolor=vermelho,plotpoints=200]{5.599999999350186E-6}{6.499999999999999}{(1.0+1.0/x)^(x)} \begin{scriptsize} \end{scriptsize} \end{pspicture*}

Figura 14. $\limitex{\infty} \left( 1+\dfrac{1}{x} \right)^ x=e$

Exercício. Calcule $\limitex{\infty} \left( 1+\dfrac{5}{x} \right)^x$.

Resolução. Fazemos a mudança de variável $t=\dfrac{x}{5}$ temos:

(86)
\[\begin{aligned} \limitex{\infty} \left( 1+\dfrac{5}{x} \right)^x& =& \limitet{\infty} \left( 1+\dfrac{1}{t} \right)^{5t}\\ & =& \left( \limitet{\infty} \left( 1+\dfrac{1}{t} \right)^{t}\right)^5\\ & =&e^5 \end{aligned} \]

Exercício. Calcule $\limitex{\infty} \left(\dfrac{x}{x+1} \right)^x$.

Resolução. Dividindo o numerador e o denominador por $x$ temos:

(87)
\[ \begin{aligned} \limitex{\infty} \left(\dfrac{x}{x+1} \right)^x & =& \limitex{\infty} \left(\dfrac{1}{1+\dfrac{1}{x}} \right)^x\\ & =& \limitex{\infty} \dfrac{1}{\left(1+\dfrac{1}{x}\right)^x} \\ & =& e^{-1} \end{aligned} \]

Logaritmo Natural

Definição 9.

O logaritmo de base $e$ é denominado função logaritmo natural ou simplesmente logaritmo. Assim a função logaritmo é a função $\ln:(0,\infty)\to \bbR$ dada pela regra

\[\ln x = y \sse e^{y}=x \]

O gráfico da função logaritmo natural está representado abaixo:

\newrgbcolor{ffwwww}{0.25 0.8 0.25} \newrgbcolor{zzzzzz}{0.6 0.6 0.6} \psset{xunit=0.56cm,yunit=0.6cm,algebraic=true,dotstyle=*,dotsize=3pt 0,linewidth=0.8pt,arrowsize=3pt 2,arrowinset=0.25} \begin{pspicture*}(-5.28,-5.24)(9.06,3.94) \psaxes[labelFontSize=\scriptstyle,xAxis=true,yAxis=true,Dx=1,Dy=1,ticksize=-2pt 0,subticks=2]{->}(0,0)(-5.28,-5.24)(9.06,3.94) \psplot[linecolor=azul,plotpoints=200]{-5.28}{9.06}{2.72^x} \psplot[plotpoints=200]{4.5600000240822146E-8}{9.06}{ln(x)} \psplot[linestyle=dotted,linecolor=zzzzzz,plotpoints=200]{-5.28}{9.06}{x} \rput[bl](-3,0.204){$e^x$} \rput[bl](7,0.6){$\ln(x)$} \end{pspicture*}

Como a função $e^x$ é contínua e crescente, pelo Teorema 13 a sua função inversa $\ln(x):(0,\infty)\to\bbR$ é contínua em todo o seu domínio.

Teorema 27.
[Terceiro Limite Fundamental]

(88)
\[\limitex{0}\dfrac{a^x-1}{x}= \ln a. \]

Demonstração. Fazendo a substituição $ u = a^h-1 $ temos que $ h = \log_a(1+u)=\dfrac{\ln(1+u)}{\ln a}$ e assim:

(89)
\[ \dfrac{e^h-1}{h} = \dfrac{u}{\ln(u+1)\cdot \ln a} = \dfrac{1}{\ln(u+1)^{\dfrac{1}{u}}}\cdot \ln a. \]

Quando $ h \to 0,\, u \to 0,$ e assim

(90)
\[ \lim_{h \to 0} \dfrac{a^h-1}{h}= \lim_{u\to 0} \dfrac{1}{\ln(u+1)^{\dfrac{1}{u}}}\cdot \ln a= \dfrac{ \ln a}{\ln e} = \ln a. \]
\scalebox{0.6}{ \psset{xunit=1.0cm,yunit=1.0cm,algebraic=true,dimen=middle,dotstyle=o,dotsize=5pt 0,linewidth=0.8pt,arrowsize=3pt 2,arrowinset=0.25} \begin{pspicture*}(-3.5,-1.5)(4.5,4.5) \psaxes[labelFontSize=\scriptstyle,xAxis=true,yAxis=true,Dx=1.,Dy=1.,ticksize=-2pt 0,subticks=2]{->}(0,0)(-3.5,-1.5)(4.5,4.5) \psplot[linewidth=1.2pt,linecolor=vermelho,plotpoints=100]{-3.5}{4.5}{(2.0^(x)-1.0)/x} \psplot[linestyle=dashed,dash=2pt 2pt,linecolor=azul]{-3.5}{4.5}{(--0.6931471805599453-0.*x)/1.} \begin{scriptsize} \rput[bl](2,3){{$\dfrac{2^x-1}{x}$}} \rput[bl](-3,1){{$\ln 2$}} \end{scriptsize} \end{pspicture*}}

Figura 15. $\limitex{0 }\dfrac{2^x-1}{x}= \ln 2.$

Exercício. Calcule o limite $\limitex{2} \frac{3^{\dfrac{x-2}{5}}-1}{x-2}$.

Resolução. Fazendo a troca de variáveis $t=\dfrac{x-2}{5}$ temos:

(91)
\[ \begin{aligned} \limitex{2} \dfrac{3^{\frac{x-2}{5}}-1}{x-2} &=& \limitet{0} \dfrac{3^t-1}{5t} &=& \dfrac{\ln 3}{5} \end{aligned} \]